2
$\begingroup$

I am investigating the perturbation of the Jordan canonical form. In my work I must calculate the number of ways to factor $p^ {n-k} q^k$ where $p$ and $q$ are distinct primes (https://oeis.org/A054225). This sequence is generated by the function: $$ \prod_{i=1}^t \prod_{j=0}^i \frac{1}{1-x^iy^j}=1+x+xy+2x^2+2x^2y+2x^2y^2+3x^3+4x^3y+4x^3y^2+3x^3y^3+\ldots. $$ I've never solved a multivariable generating function before. Could you advise please, how to prove that the function is generated for the sequence, or advise the literature on this subject?

Any help would be greatly appreciated.

$\endgroup$
4
  • 1
    $\begingroup$ What does "solve" mean for you? You can't expect a simple formula. $\endgroup$ Dec 1, 2013 at 18:07
  • $\begingroup$ I'm not sure if this is what you want, but the oeis page also gives some programs (maple, haskell, pari, mathematica) to compute the elements of the sequence. $\endgroup$
    – guest
    Dec 1, 2013 at 20:20
  • $\begingroup$ I would like to know the technique of proving that the sequence is generated by the function. $\endgroup$
    – Alexander
    Dec 2, 2013 at 5:20
  • $\begingroup$ what are $t$ and $i$ in the product limits? $\endgroup$ Nov 20, 2019 at 22:47

1 Answer 1

1
$\begingroup$

The given g.f. directly follows from the sequence definition.

Indeed, the sequence gives the number of partitions of vector $(n,m)$ into the sum of vectors with nonnegative integer components. Each possible vector $(i,j)$ is encoded by the term $$\frac{1}{1-x^iy^j} = 1 + x^iy^j + x^{2i}y^{2j} + x^{3i}y^{3j} + \dots,$$ where the summand $x^{ki}y^{kj}$ encodes the contribution of this vector when it is taken $k$ times (the powers of $x$ and $y$ stand for the first and second component of $k\cdot (i,j)$, respectively). Then, the total contribution of all vectors in a partition results in $x^ny^m$, and the coefficient of this monomial in the product gives the number of different partitions.

$\endgroup$
1
  • 2
    $\begingroup$ These are sometimes called "vector partitions" or "bipartite partitions". $\endgroup$
    – Ira Gessel
    Nov 21, 2019 at 0:39

Your Answer

By clicking “Post Your Answer”, you agree to our terms of service and acknowledge that you have read and understand our privacy policy and code of conduct.

Not the answer you're looking for? Browse other questions tagged or ask your own question.